LSAT and Law School Admissions Forum

Get expert LSAT preparation and law school admissions advice from PowerScore Test Preparation.

User avatar
 Dave Killoran
PowerScore Staff
  • PowerScore Staff
  • Posts: 5852
  • Joined: Mar 25, 2011
|
#27096
Complete Question Explanation
(The complete setup for this game can be found here: lsat/viewtopic.php?f=325&t=6210)

The correct answer choice is (D)

The condition in the question stem establishes that the planting committee has exactly three members: G, H, and L. The trails committee can also have these three members, which maximizes the number of members in common. But, the trails committee must also have M as a member in order to satisfy the fourth rule, leading to the following setup for this question:
June 98_M12_game#1_L6_explanations_game#1_#3_diagram_1.png
Note that the setup for the trails committee contains the minimum number of members; there could be other members of the trails committee. The planting committee is complete.

Answer choices (A), (B), and (C) are incorrect because the question stem stipulates that the two committees have “as many members in common as the conditions allow,” which in this instance is three members.

Answer choice (D) is the correct answer choice
, and is proven by the setup to the question.

Answer choice (E) is incorrect because this answer would force the trails committee to have just one member, a violation of the first rule.

Get the most out of your LSAT Prep Plus subscription.

Analyze and track your performance with our Testing and Analytics Package.